Non-Relativistic Limit of Klein-Gordon Probability Density












3












$begingroup$


In the lecture notes accompanying an introductory course in relativistic quantum mechanics, the Klein-Gordon probability density and current are defined as:
$$
begin{eqnarray}
P & = & dfrac{ihbar}{2mc^2}left(Phi^*dfrac{partialPhi}{partial t}-Phidfrac{partialPhi^*}{partial t}right) \
vec{j} &=& dfrac{hbar}{2mi}left(Phi^*vec{nabla}Phi-Phivec{nabla}Phi^*right)
end{eqnarray}
$$

together with the statement that:




One can show that in the non-relativistic limit, the known expressions for the probability density and current are recovered.




The 'known' expressions are:
$$
begin{eqnarray}
rho &=& Psi^*Psi \
vec{j} &=& dfrac{hbar}{2mi}left(Psi^*vec{nabla}Psi-Psivec{nabla}Psi^*right)
end{eqnarray}
$$



When taking a 'non-relativistic limit', I am used to taking the limit $c to infty$, which does give the right result for $vec{j}$, but for the density produces $P=0$. How should one then take said limit to recover the non-relativistic equations?










share|cite|improve this question











$endgroup$








  • 1




    $begingroup$
    For a connection between Schr. eq. and Klein-Gordon eq, see e.g. A. Zee, QFT in a Nutshell, Chap. III.5, and this Phys.SE post plus links therein.
    $endgroup$
    – Qmechanic
    Dec 23 '18 at 21:39
















3












$begingroup$


In the lecture notes accompanying an introductory course in relativistic quantum mechanics, the Klein-Gordon probability density and current are defined as:
$$
begin{eqnarray}
P & = & dfrac{ihbar}{2mc^2}left(Phi^*dfrac{partialPhi}{partial t}-Phidfrac{partialPhi^*}{partial t}right) \
vec{j} &=& dfrac{hbar}{2mi}left(Phi^*vec{nabla}Phi-Phivec{nabla}Phi^*right)
end{eqnarray}
$$

together with the statement that:




One can show that in the non-relativistic limit, the known expressions for the probability density and current are recovered.




The 'known' expressions are:
$$
begin{eqnarray}
rho &=& Psi^*Psi \
vec{j} &=& dfrac{hbar}{2mi}left(Psi^*vec{nabla}Psi-Psivec{nabla}Psi^*right)
end{eqnarray}
$$



When taking a 'non-relativistic limit', I am used to taking the limit $c to infty$, which does give the right result for $vec{j}$, but for the density produces $P=0$. How should one then take said limit to recover the non-relativistic equations?










share|cite|improve this question











$endgroup$








  • 1




    $begingroup$
    For a connection between Schr. eq. and Klein-Gordon eq, see e.g. A. Zee, QFT in a Nutshell, Chap. III.5, and this Phys.SE post plus links therein.
    $endgroup$
    – Qmechanic
    Dec 23 '18 at 21:39














3












3








3


1



$begingroup$


In the lecture notes accompanying an introductory course in relativistic quantum mechanics, the Klein-Gordon probability density and current are defined as:
$$
begin{eqnarray}
P & = & dfrac{ihbar}{2mc^2}left(Phi^*dfrac{partialPhi}{partial t}-Phidfrac{partialPhi^*}{partial t}right) \
vec{j} &=& dfrac{hbar}{2mi}left(Phi^*vec{nabla}Phi-Phivec{nabla}Phi^*right)
end{eqnarray}
$$

together with the statement that:




One can show that in the non-relativistic limit, the known expressions for the probability density and current are recovered.




The 'known' expressions are:
$$
begin{eqnarray}
rho &=& Psi^*Psi \
vec{j} &=& dfrac{hbar}{2mi}left(Psi^*vec{nabla}Psi-Psivec{nabla}Psi^*right)
end{eqnarray}
$$



When taking a 'non-relativistic limit', I am used to taking the limit $c to infty$, which does give the right result for $vec{j}$, but for the density produces $P=0$. How should one then take said limit to recover the non-relativistic equations?










share|cite|improve this question











$endgroup$




In the lecture notes accompanying an introductory course in relativistic quantum mechanics, the Klein-Gordon probability density and current are defined as:
$$
begin{eqnarray}
P & = & dfrac{ihbar}{2mc^2}left(Phi^*dfrac{partialPhi}{partial t}-Phidfrac{partialPhi^*}{partial t}right) \
vec{j} &=& dfrac{hbar}{2mi}left(Phi^*vec{nabla}Phi-Phivec{nabla}Phi^*right)
end{eqnarray}
$$

together with the statement that:




One can show that in the non-relativistic limit, the known expressions for the probability density and current are recovered.




The 'known' expressions are:
$$
begin{eqnarray}
rho &=& Psi^*Psi \
vec{j} &=& dfrac{hbar}{2mi}left(Psi^*vec{nabla}Psi-Psivec{nabla}Psi^*right)
end{eqnarray}
$$



When taking a 'non-relativistic limit', I am used to taking the limit $c to infty$, which does give the right result for $vec{j}$, but for the density produces $P=0$. How should one then take said limit to recover the non-relativistic equations?







quantum-mechanics wavefunction speed-of-light schroedinger-equation klein-gordon-equation






share|cite|improve this question















share|cite|improve this question













share|cite|improve this question




share|cite|improve this question








edited Dec 23 '18 at 21:37









Qmechanic

103k121861182




103k121861182










asked Dec 23 '18 at 21:21









SimonSimon

771313




771313








  • 1




    $begingroup$
    For a connection between Schr. eq. and Klein-Gordon eq, see e.g. A. Zee, QFT in a Nutshell, Chap. III.5, and this Phys.SE post plus links therein.
    $endgroup$
    – Qmechanic
    Dec 23 '18 at 21:39














  • 1




    $begingroup$
    For a connection between Schr. eq. and Klein-Gordon eq, see e.g. A. Zee, QFT in a Nutshell, Chap. III.5, and this Phys.SE post plus links therein.
    $endgroup$
    – Qmechanic
    Dec 23 '18 at 21:39








1




1




$begingroup$
For a connection between Schr. eq. and Klein-Gordon eq, see e.g. A. Zee, QFT in a Nutshell, Chap. III.5, and this Phys.SE post plus links therein.
$endgroup$
– Qmechanic
Dec 23 '18 at 21:39




$begingroup$
For a connection between Schr. eq. and Klein-Gordon eq, see e.g. A. Zee, QFT in a Nutshell, Chap. III.5, and this Phys.SE post plus links therein.
$endgroup$
– Qmechanic
Dec 23 '18 at 21:39










2 Answers
2






active

oldest

votes


















3












$begingroup$

The trick is to make the approach for the relativistic Klein-Gordon wave function
$$ Phi(vec{r},t) = Psi(vec{r},t) e^{-imc^2t/hbar} $$
(The physical reasoning behind this approach is:
The fast oscillating exponential is the solution for the particle at rest.
And compared to that, $Psi$ will give only slow variations.)



From that you find its derivatives
$$
begin{eqnarray}
frac{partialPhi}{partial t} &=& left( frac{partialPsi}{partial t} - frac{imc^2}{hbar}Psi right) e^{-imc^2t/hbar} \
vec{nabla}Phi &=& vec{nabla}Psi e^{-imc^2t/hbar}
end{eqnarray}
$$



Plug these into the definitions of the Klein-Gordon probability density and current ($P$ and $vec{j}$) and you get
$$
begin{eqnarray}
P &=& Psi^* Psi + frac{ihbar}{2mc^2} left( Psi^*frac{partialPsi}{partial t} - Psi frac{partialPsi^*}{partial t} right) \
vec{j} &=& dfrac{hbar}{2mi}left(Psi^*vec{nabla}Psi-Psivec{nabla}Psi^*right)
end{eqnarray}
$$



Now you can do the limit $c to infty$ and get the 'known' expressions of the non-relativistic Schrödinger probability density and current.






share|cite|improve this answer











$endgroup$





















    2












    $begingroup$

    You can substitute $Phi = e^{-mc^2t/hbar} Psi$ and then neglect the second order time derivative of $Psi$. Drop the constant $mc^2$ and you will have recovered the Schrödinger equation.






    share|cite|improve this answer









    $endgroup$













      Your Answer





      StackExchange.ifUsing("editor", function () {
      return StackExchange.using("mathjaxEditing", function () {
      StackExchange.MarkdownEditor.creationCallbacks.add(function (editor, postfix) {
      StackExchange.mathjaxEditing.prepareWmdForMathJax(editor, postfix, [["$", "$"], ["\\(","\\)"]]);
      });
      });
      }, "mathjax-editing");

      StackExchange.ready(function() {
      var channelOptions = {
      tags: "".split(" "),
      id: "151"
      };
      initTagRenderer("".split(" "), "".split(" "), channelOptions);

      StackExchange.using("externalEditor", function() {
      // Have to fire editor after snippets, if snippets enabled
      if (StackExchange.settings.snippets.snippetsEnabled) {
      StackExchange.using("snippets", function() {
      createEditor();
      });
      }
      else {
      createEditor();
      }
      });

      function createEditor() {
      StackExchange.prepareEditor({
      heartbeatType: 'answer',
      autoActivateHeartbeat: false,
      convertImagesToLinks: false,
      noModals: true,
      showLowRepImageUploadWarning: true,
      reputationToPostImages: null,
      bindNavPrevention: true,
      postfix: "",
      imageUploader: {
      brandingHtml: "Powered by u003ca class="icon-imgur-white" href="https://imgur.com/"u003eu003c/au003e",
      contentPolicyHtml: "User contributions licensed under u003ca href="https://creativecommons.org/licenses/by-sa/3.0/"u003ecc by-sa 3.0 with attribution requiredu003c/au003e u003ca href="https://stackoverflow.com/legal/content-policy"u003e(content policy)u003c/au003e",
      allowUrls: true
      },
      noCode: true, onDemand: true,
      discardSelector: ".discard-answer"
      ,immediatelyShowMarkdownHelp:true
      });


      }
      });














      draft saved

      draft discarded


















      StackExchange.ready(
      function () {
      StackExchange.openid.initPostLogin('.new-post-login', 'https%3a%2f%2fphysics.stackexchange.com%2fquestions%2f450076%2fnon-relativistic-limit-of-klein-gordon-probability-density%23new-answer', 'question_page');
      }
      );

      Post as a guest















      Required, but never shown

























      2 Answers
      2






      active

      oldest

      votes








      2 Answers
      2






      active

      oldest

      votes









      active

      oldest

      votes






      active

      oldest

      votes









      3












      $begingroup$

      The trick is to make the approach for the relativistic Klein-Gordon wave function
      $$ Phi(vec{r},t) = Psi(vec{r},t) e^{-imc^2t/hbar} $$
      (The physical reasoning behind this approach is:
      The fast oscillating exponential is the solution for the particle at rest.
      And compared to that, $Psi$ will give only slow variations.)



      From that you find its derivatives
      $$
      begin{eqnarray}
      frac{partialPhi}{partial t} &=& left( frac{partialPsi}{partial t} - frac{imc^2}{hbar}Psi right) e^{-imc^2t/hbar} \
      vec{nabla}Phi &=& vec{nabla}Psi e^{-imc^2t/hbar}
      end{eqnarray}
      $$



      Plug these into the definitions of the Klein-Gordon probability density and current ($P$ and $vec{j}$) and you get
      $$
      begin{eqnarray}
      P &=& Psi^* Psi + frac{ihbar}{2mc^2} left( Psi^*frac{partialPsi}{partial t} - Psi frac{partialPsi^*}{partial t} right) \
      vec{j} &=& dfrac{hbar}{2mi}left(Psi^*vec{nabla}Psi-Psivec{nabla}Psi^*right)
      end{eqnarray}
      $$



      Now you can do the limit $c to infty$ and get the 'known' expressions of the non-relativistic Schrödinger probability density and current.






      share|cite|improve this answer











      $endgroup$


















        3












        $begingroup$

        The trick is to make the approach for the relativistic Klein-Gordon wave function
        $$ Phi(vec{r},t) = Psi(vec{r},t) e^{-imc^2t/hbar} $$
        (The physical reasoning behind this approach is:
        The fast oscillating exponential is the solution for the particle at rest.
        And compared to that, $Psi$ will give only slow variations.)



        From that you find its derivatives
        $$
        begin{eqnarray}
        frac{partialPhi}{partial t} &=& left( frac{partialPsi}{partial t} - frac{imc^2}{hbar}Psi right) e^{-imc^2t/hbar} \
        vec{nabla}Phi &=& vec{nabla}Psi e^{-imc^2t/hbar}
        end{eqnarray}
        $$



        Plug these into the definitions of the Klein-Gordon probability density and current ($P$ and $vec{j}$) and you get
        $$
        begin{eqnarray}
        P &=& Psi^* Psi + frac{ihbar}{2mc^2} left( Psi^*frac{partialPsi}{partial t} - Psi frac{partialPsi^*}{partial t} right) \
        vec{j} &=& dfrac{hbar}{2mi}left(Psi^*vec{nabla}Psi-Psivec{nabla}Psi^*right)
        end{eqnarray}
        $$



        Now you can do the limit $c to infty$ and get the 'known' expressions of the non-relativistic Schrödinger probability density and current.






        share|cite|improve this answer











        $endgroup$
















          3












          3








          3





          $begingroup$

          The trick is to make the approach for the relativistic Klein-Gordon wave function
          $$ Phi(vec{r},t) = Psi(vec{r},t) e^{-imc^2t/hbar} $$
          (The physical reasoning behind this approach is:
          The fast oscillating exponential is the solution for the particle at rest.
          And compared to that, $Psi$ will give only slow variations.)



          From that you find its derivatives
          $$
          begin{eqnarray}
          frac{partialPhi}{partial t} &=& left( frac{partialPsi}{partial t} - frac{imc^2}{hbar}Psi right) e^{-imc^2t/hbar} \
          vec{nabla}Phi &=& vec{nabla}Psi e^{-imc^2t/hbar}
          end{eqnarray}
          $$



          Plug these into the definitions of the Klein-Gordon probability density and current ($P$ and $vec{j}$) and you get
          $$
          begin{eqnarray}
          P &=& Psi^* Psi + frac{ihbar}{2mc^2} left( Psi^*frac{partialPsi}{partial t} - Psi frac{partialPsi^*}{partial t} right) \
          vec{j} &=& dfrac{hbar}{2mi}left(Psi^*vec{nabla}Psi-Psivec{nabla}Psi^*right)
          end{eqnarray}
          $$



          Now you can do the limit $c to infty$ and get the 'known' expressions of the non-relativistic Schrödinger probability density and current.






          share|cite|improve this answer











          $endgroup$



          The trick is to make the approach for the relativistic Klein-Gordon wave function
          $$ Phi(vec{r},t) = Psi(vec{r},t) e^{-imc^2t/hbar} $$
          (The physical reasoning behind this approach is:
          The fast oscillating exponential is the solution for the particle at rest.
          And compared to that, $Psi$ will give only slow variations.)



          From that you find its derivatives
          $$
          begin{eqnarray}
          frac{partialPhi}{partial t} &=& left( frac{partialPsi}{partial t} - frac{imc^2}{hbar}Psi right) e^{-imc^2t/hbar} \
          vec{nabla}Phi &=& vec{nabla}Psi e^{-imc^2t/hbar}
          end{eqnarray}
          $$



          Plug these into the definitions of the Klein-Gordon probability density and current ($P$ and $vec{j}$) and you get
          $$
          begin{eqnarray}
          P &=& Psi^* Psi + frac{ihbar}{2mc^2} left( Psi^*frac{partialPsi}{partial t} - Psi frac{partialPsi^*}{partial t} right) \
          vec{j} &=& dfrac{hbar}{2mi}left(Psi^*vec{nabla}Psi-Psivec{nabla}Psi^*right)
          end{eqnarray}
          $$



          Now you can do the limit $c to infty$ and get the 'known' expressions of the non-relativistic Schrödinger probability density and current.







          share|cite|improve this answer














          share|cite|improve this answer



          share|cite|improve this answer








          edited Jan 7 at 23:38

























          answered Dec 24 '18 at 0:54









          Thomas FritschThomas Fritsch

          34929




          34929























              2












              $begingroup$

              You can substitute $Phi = e^{-mc^2t/hbar} Psi$ and then neglect the second order time derivative of $Psi$. Drop the constant $mc^2$ and you will have recovered the Schrödinger equation.






              share|cite|improve this answer









              $endgroup$


















                2












                $begingroup$

                You can substitute $Phi = e^{-mc^2t/hbar} Psi$ and then neglect the second order time derivative of $Psi$. Drop the constant $mc^2$ and you will have recovered the Schrödinger equation.






                share|cite|improve this answer









                $endgroup$
















                  2












                  2








                  2





                  $begingroup$

                  You can substitute $Phi = e^{-mc^2t/hbar} Psi$ and then neglect the second order time derivative of $Psi$. Drop the constant $mc^2$ and you will have recovered the Schrödinger equation.






                  share|cite|improve this answer









                  $endgroup$



                  You can substitute $Phi = e^{-mc^2t/hbar} Psi$ and then neglect the second order time derivative of $Psi$. Drop the constant $mc^2$ and you will have recovered the Schrödinger equation.







                  share|cite|improve this answer












                  share|cite|improve this answer



                  share|cite|improve this answer










                  answered Dec 24 '18 at 0:32









                  my2ctsmy2cts

                  4,9422618




                  4,9422618






























                      draft saved

                      draft discarded




















































                      Thanks for contributing an answer to Physics Stack Exchange!


                      • Please be sure to answer the question. Provide details and share your research!

                      But avoid



                      • Asking for help, clarification, or responding to other answers.

                      • Making statements based on opinion; back them up with references or personal experience.


                      Use MathJax to format equations. MathJax reference.


                      To learn more, see our tips on writing great answers.




                      draft saved


                      draft discarded














                      StackExchange.ready(
                      function () {
                      StackExchange.openid.initPostLogin('.new-post-login', 'https%3a%2f%2fphysics.stackexchange.com%2fquestions%2f450076%2fnon-relativistic-limit-of-klein-gordon-probability-density%23new-answer', 'question_page');
                      }
                      );

                      Post as a guest















                      Required, but never shown





















































                      Required, but never shown














                      Required, but never shown












                      Required, but never shown







                      Required, but never shown

































                      Required, but never shown














                      Required, but never shown












                      Required, but never shown







                      Required, but never shown







                      Popular posts from this blog

                      Список кардиналов, возведённых папой римским Каликстом III

                      Deduzione

                      Mysql.sock missing - “Can't connect to local MySQL server through socket”